English Deutsch Français Italiano Español Português 繁體中文 Bahasa Indonesia Tiếng Việt ภาษาไทย
All categories

I have a pretty good grasp of integration but don't get this question! Thanks!

2006-06-25 20:46:01 · 7 answers · asked by Anonymous in Science & Mathematics Mathematics

7 answers

To perhaps elaborate on nardneran's answer:

In this problem you use the substitution rule. Let u=ln x. Then du=1/x dx and dx=x du. Replacing ln x with u and dx with x du gives ∫u² du (the x's in your equation cancel). You can integrate that to get u³/3 + C and back-substituting gives (ln x)³/3 + C.

2006-06-25 22:08:54 · answer #1 · answered by Pascal 7 · 0 1

Let u=ln(x). Then du=(1/x) dx, so you get the integral of u^2 du.

2006-06-26 07:35:23 · answer #2 · answered by mathematician 7 · 0 0

integral of ln(x)^2/x is very very simple...

its intergral ln(x)^2/x dx [ and 1/x dx can be written as d( ln(x) ) ]

so its of the form f(x) d( f(x) )

so reduces to Y^2 dY where Y = ln(x)

there fore the answer is ln(x)^3/3 + C

2006-06-26 03:58:08 · answer #3 · answered by nardneran 1 · 0 0

considering the ques. take log x =t
differentiate both sides
1/x= dt/dx

y= t^2 dt/dx

now integrate
y= 1/3 t^3 + c

y= 1/3 (logx)^3 +c

2006-06-26 06:15:46 · answer #4 · answered by Anonymous · 0 0

If you differentiate your answer, you will see why. The +C is obviously required as a constant of integration as you do not have limits for x expressed in the question.

2006-06-26 05:26:38 · answer #5 · answered by the_dt 4 · 0 0

AHHHHHHHHHHH..Damn..Now my brain hurts... I knew I shouldnt have clicked on the mathmatics section!

2006-06-26 03:58:50 · answer #6 · answered by Greg W 2 · 0 0

its the quotient rule...

2006-06-26 03:50:11 · answer #7 · answered by Jesters Deadd 2 · 0 0

fedest.com, questions and answers